darla has 3 times as many penciled as lucy. darla had 27 how many pencils does lucy have .

Answers

Answer 1

Answer:

9

Step-by-step explanation:

just do 27 divided by 3

Answer 2

Answer:

9

Step-by-step explanation:

9 × 3 = 27 or 27 ÷ 3 = 9


Related Questions

Which number line correctly shows –3 – 1.5?
A number line going from negative 4.5 to positive 4.5. An arrow goes from 0 to negative 3 and from negative 3 to negative 4.5.
A number line going from negative 4.5 to positive 4.5. An arrow goes from 0 to 3 and from 3 to 4.5.
A number line going from negative 4.5 to positive 4.5. An arrow goes from negative 3 to negative 1.5 and from 0 to negative 3.
A number line going from negative 4.5 to positive 4.5. An arrow goes from negative 1.5 to 1.5 and from 0 to negative 1.5..

Answers

Answer:5.4

Step-by-step explanation:


List the names and measures for pairs of alternate Interior angles. Group them together. Then list the names and measures for pairs of alternate
exterior angles.

Answers

Answer:

The two pairs of alternate interior angles are CHG and HGB and DHG and HGA.

mCHG  =  mHGB  =  48.81°

mDHG  =  mHGA  =  131.19°

The two pairs of alternate exterior angles are EHC and BGF and EHD and AGF.

mEHC  =  mBGF  =  131.19°

mEHD  =  mAGF  =  48.81°

Step-by-step explanation:

STRAIHGT FROM PLATO BOOOOIIIIII

This is probably easy, but please help me​

Answers

I would guess if that’s fine

Bob works as a plumber. He charges $45.00 for the first hour and $33.75 for each additional hour. Bob was paid $247.50 for his last job
How many hours did Bob work on his last job?

Answers

Answer:

7 hours

Step-by-step explanation:

$247.50 - $45.00 (how much he charges for the first hour) = $202.50

$202.50 ÷ $33.75 (how much he charges for the additional hours) = 6 hours

But he charges 45 dollars for the first hour so he 7 hours in total

slope=?

please help

Answers

Answer: It's 4

Step-by-step explanation: 12-8 is 4, 4-3 is 1

Find the quotient.
36s^3t-26st
-----------------
-2st
Enter the correct answer.

Answers

Answer:

it is makeing me do it

Step-by-step explanation:

PLEASE ANSWER IM TIMED Use the graph above to write down an equation and fill out the table. Identify the charge for each hour of work and the one-time fee

Answers

Answer:

y= 50x + 150

charge for each hour is 50

one time fee is 150

Step-by-step explanation:

Mathematical Connections Jake's solution to
the equation - 4(2x - 3) = 36 is shown.
-4(2x - 3) = 36
-8x + 12 - 36
- 8x + 12 - 12 = 36 - 12
-8x = 24
8x24
-8
X=-3
How is the solution to -4(2x - 3) > 36 similar to
and different from the solution shown?

Answers

6739-77(7) =67997 +66

The solution x > -3 represents all values of x that are greater than -3. On the other hand, the solution x = -3 represents a single value of x that is equal to -3.

Given that,

An equation is,

- 4(2x - 3) = 36

Here, Jake's solution to the equation - 4(2x - 3) = 36 is,

-4(2x - 3) = 36

Apply distributive property,

-8x + 12 - 36

- 8x + 12 - 12 = 36 - 12

Subtract 12 on both sides,

-8x = 24

x = - 24/8

x = - 3

So, the solution x = -3 represents a single value of x that is equal to -3.

Therefore, x > -3 represents a range of values greater than -3, while x = -3 represents a single value that is equal to -3.

To learn more about the equation visit:

brainly.com/question/28871326

#SPJ6

could someone out there perhaps help me? i’m so lost even though it’s probably so simple. i’ll give you brainleist :)

Answers

Answer:

y= -x+7, b= sqrt(2P/a), c=3P^2-b

Step-by-step explanation:

First, make a table regarding both of the equations. You will eventually find out that both lines intersect at the point (2, 5) after you find the points on the table. From there, subtract x from both sides in the equation x + y = 2. You will      get y = -x + 2. Since they said the line was parallel, find a line that has the slope of negative one. Since we know that this line intersects the point in which the first two lines intersect, we know that the y-intercept will be 7. The equation of the line would be y=-x+7.

Multiply both sides by 2. Then, divide both sides by a to get b^2=(2P/a). Take the square root to get the value of b, which is sqrt(2P/a).

Square both sides of the equation to get P^2=(b+c)/3. Cross multiply to get 3P^2=b+c. Subtract b from both sides to get c=3P^2-b.

math help please! - ( 2/5 )

Answers

Answer:

D

Step-by-step explanation:

it is D because you always want x to be a lone fraction and that would be the first step im 99% sure unless someone can correct me :)

Answer:

d) subtract 15 from both sides

Step-by-step explanation:

Process of elimination:

a) 2.5 and x are being multiplied, not subtracted, so (a) is wrong right off the bat.

b) you may divide 2.5 by both sides, but you want to get "x" on one side first. So you would do (b) first either.

c) to get rid of 15, which we need to somehow, we need to do the opposite operation than the one in the equation. 15 is +15, not negative, and so you would need to subtract to get rid of 15. This is why (c) is also wrong.

That leaves us, with (d), because you need to subtract 15 to isolate "x".

PLEASE HELP Due today!

What properties are these?

1. if PQ = PQ, then line PQ≅line PQ.
4. If RS = TU, then RS + XY = TU + XY
5. If AB = DE, then DE =AB
6. If Y is the midpoint of line XZ, then XY = YZ
7. if line FG ≅ line HI and line HI ≅ line JK, then line FG ≅ line JK
8. If AB+CD=EF+CD, then AB=EF
9. If PQ+RS=TV and RS=WX, then PQ+WX=TV
10. If LP=PN, and L, P, and N are collinear, then P is the midpoint of line LN
11. If line UV ≅ line UV, then UV=UV

Property bank:
Addition Property Of Equality
Multiplication Property Of Equality
Division Property Of Equality
Distributive Property Of Equality
Substitution Property Of Equality
Symmetric Property Of Equality
Transitive Property Of Equality
Reflexive Property Of Congruence
Symmetric Property Of Congruence
Transitive Property Of Congruence
Definition of Congruence
Definition of Midpoint

Answers

Answer:

it looks like it is Addition Property Of Equality

Step-by-step explanation:

How many solutions does this have? 2x = 2x - 9
A. No
B. One
C. Two
D. Infinity

Answers

Answer:

A. No solutions

Step-by-step explanation:

2x-2x=-9

0= -9

Since 0 does not equal -9, there are no solutions.

A:there is no solution

Answer the questions using the drop-down menus.
At what type of boundary do strike-strip basins form?
At what type of boundary do wedge basins form?
At what type of boundary do rift basins form?
At what type of boundary do arc basins form?

Answers

Answer:  

At what type of boundary do strike-strip basins form? transform

At what type of boundary do wedge basins form? convergent

At what type of boundary do rift basins form? divergent

At what type of boundary do arc basins form? convergent

Step-by-step explanation:

Hope this helps!

Answer:

Answer the questions using the drop-down menus.

✔ transform

At what type of boundary do wedge basins form?

✔ convergent

At what type of boundary do rift basins form?

✔ divergent

At what type of boundary do arc basins form?

✔ convergent

Step-by-step explanation:

Which set of expressions are not equivalent? (4 1/2a + 2) + 3 and 5 + 4 1/2 a

Answers

Answer:

not equivalent

Step-by-step explanation:

2a+11

5+9/2a

3/4(2x+1)=2 solve for x

Answers

x= 5/6 (or .83 if you'd prefer it in decimal form.)

Answer:

3/4(2x+1)=2

3/2x+3/4=2

3/2x=2 -3/4

3/2x=-10/4

3/2x(2)=-10/4(2)

3x= -5/2

3x/3 = -5/2 * 1/3

x= -5/6

If eight gallons of gasoline costs $21.44, what is the
cost for fifteen gallons of gasoline?
Equation:
Solution:

Answers

Answer:

$40.2 dollars

Please make me brainliest!

Hurry someone please answer plsss
Find the midpoint of each line segment.

Answers

Answer:

2.5

Step-by-step explanation:

you can use pythag theorem and divide c by two:

-3^2+4^2=c^2

9+16= 25

c=5

however, 5/2=2.5

how to solve −3/4n≤3.

Answers

Answer:

n ≥ -4

Step-by-step explanation:

-3/4n ≤ 3

-4/3(-3/4n) ≤ 3(-4/3)

n ≥ -12/3

n ≥ -4

CHECK:

correct

-3/4(-4) ≤ 3

12/4 ≤ 3

3 = 3

correct

-3/4(5) ≤ 3

-15/4 ≤ 3

-3.75 < 3

incorrect

-3/4(1) ≤ 3

-3/4 is not ≤ 3

Find the missing angle measures

Thank you!!

Answers

Answer:

SEE BELOW

Step-by-step explanation:

INTERRIOR ANGLES

3.

∠A + ∠B + ∠C = 180

(180 - 103) + (3x - 14) + (3x + 9) = 180

6x = 180 - 77 + 14 - 9

x = 108 / 6

x = 18

m∠ACB = (3x - 14)

             = 3(18) - 14

             = 40

m∠ABC = (3x + 9)

              = 3(18) + 9

              = 63

-------------------------------

INTERRIOR ANGLES

4.

∠A + ∠B + ∠C = 180

(180 - 97) + (2x + 2) + (14x - 1) = 180

16x = 180 - 83 - 2 + 1

x = 96 / 16

x = 6

m∠BCA = 2x + 2

             = 2(6) + 2

             = 14

m∠ABC = 14x - 1

             = 14(6) - 1

             = 83

-------------------------------

EXTERIOR ANGLES

5.

∠B

180 - 15x + 20

15x = 180 + 20

   x = 200/15

   x = 13.33

∠C

180 - 9x + 10  

9x = 190

 x = 190/9

 x = 21.11

∠E

180 - 17x + 2

17x = 182

  x = 182/17

  x = 10.706

m∠BEF = 17x + 2

             = 17 (10.706) + 2

             = 184

m∠ABC = 15x + 20

             = 15(15.33) + 20

             = 220

-------------------------------

EXTERIOR ANGLES

6.

first solve x by adding all the interior angles = 180

∠B + ∠E + ∠C = 180

2x + x + 93 = 180

3x = 180 - 93

x = 87/2

x = 43.5

m∠BEC = 360 - x

             = 360 - 43.5

             = 316.5

m∠ABC = 180 - 2x

             = 180 - 2(43.5)

             = 93

Simplify negative 4 over 5 ÷ 3 over negative 2.
The answer choices are:
negative 8 over 15
negative 6 over 5
6 over 5
8 over 15

Answers

Answer: 8/15

Step-by-step explanation:

(-4/5) divided by (-3/2) becomes (-4/5) x (-2/3) because when dividing fractions, change it to multiplication and flip the divider (in this case -3/2)

Multiply numerators to get 8 and denominators to get 15 resulting in 8/15

Keith has 500 in his bank account. He earns $25 per week. He wants to have at least $740 in his Account at the end of the summer. How many weeks Miss Keith work?

Answers

Answer:

9 more weeks

Step-by-step explanation:

because if you do 740-500 you get 240

so then i divide 240 by the money per week (25) and

get 9.6 so it is 9 weeks

Slope = 2; y-intercept = -1

Answers

Answer: y=2x-1

Step-by-step explanation:

In slope intercept form y=Mx+b

Mx=slope and b=intercept

Y=2x-1

The equation of line with slope m = 2 and y-intercept is -1 is given by

y = 2x - 1

What is an Equation of a line?

The equation of a line is expressed as y = mx + b where m is the slope and b is the y-intercept

And y - y₁ = m ( x - x₁ )

y = y-coordinate of second point

y₁ = y-coordinate of point one

m = slope

x = x-coordinate of second point

x₁ = x-coordinate of point one

The slope m = ( y₂ - y₁ ) / ( x₂ - x₁ )

Given data ,

Let the equation of line be represented as A

Now , the value of A is

Let the slope of the line m = 2

Let the y intercept of the line b = -1

Now , equation of a line is expressed as y = mx + b where m is the slope and b is the y-intercept

Substituting the values in the equation , we get

y = 2x - 1   be equation (1)

Therefore , the value of A is y = 2x - 1

Hence , the equation of line is y = 2x - 1

To learn more about equation of line click :

https://brainly.com/question/14200719

#SPJ5

There are 12 students in the gym class. If 4 students can sit on 1 bench, how many benches are needed for all students ?

Answers

Answer:

3 benches needed

Step-by-step explanation:

Answer:

3 benches

Step-by-step explanation:

4 people to one bench

12 people divided by 4 = 3 benches

PLZ ANSWER WILL GIVE BRAINLIEST TO CORRECT ANSWER!!!
Which expression represents the correct factorization of the following polynomial?
27x^3+64

A.(3x+4)(3x^2−12x+64)

B.3x+4(9x^2+12x−16)

C.(3x+4)(9x^2−12x+16)

D.(9x+16)(3x^2−12x+4)

Answers

Answer:

I'm pretty sure that this one is the correct factorization of the polynomial above: c) (3x+4)(9x^2-12x+16)

(3x−4)⋅(9x  

2

+12x+16)

See steps

Step by Step Solution:

More Icon

STEP

1

:

Equation at the end of step 1

 33x3 -  64

STEP

2

:

Trying to factor as a Difference of Cubes

2.1      Factoring:  27x3-64  

Theory : A difference of two perfect cubes,  a3 - b3 can be factored into

             (a-b) • (a2 +ab +b2)

Proof :  (a-b)•(a2+ab+b2) =

           a3+a2b+ab2-ba2-b2a-b3 =

           a3+(a2b-ba2)+(ab2-b2a)-b3 =

           a3+0+0+b3 =

           a3+b3

Check :  27  is the cube of  3  

Check :  64  is the cube of   4  

Check :  x3 is the cube of   x1

Factorization is :

            (3x - 4)  •  (9x2 + 12x + 16)  

Trying to factor by splitting the middle term

2.2     Factoring  9x2 + 12x + 16  

The first term is,  9x2  its coefficient is  9 .

The middle term is,  +12x  its coefficient is  12 .

The last term, "the constant", is  +16  

Step-1 : Multiply the coefficient of the first term by the constant   9 • 16 = 144  

Step-2 : Find two factors of  144  whose sum equals the coefficient of the middle term, which is   12 .

     -144    +    -1    =    -145  

     -72    +    -2    =    -74  

     -48    +    -3    =    -51  

     -36    +    -4    =    -40  

     -24    +    -6    =    -30  

     -18    +    -8    =    -26  

For tidiness, printing of 24 lines which failed to find two such factors, was suppressed

Observation : No two such factors can be found !!

Conclusion : Trinomial can not be factored

Final result :

 (3x - 4) • (9x2 + 12x + 16)

Answer:

C

Step-by-step explanation:

When factorising the sum of cubes, we use the formula

a^3+b^3=(a+b)(a^2−ab+b^2).

In this case of 27x3+64,

27x^3=a^3

64=b^3

Find a:

27x^3=a^3

3^√27x^3=3^√a^3

3x=a

Find b:

64=b^3

3^√64=3√b^3

4=b

Substitute a=3x and b=4 into

 (a+b)(a^2−ab+b^2)

(3x+4)((3x)^2−(3x×4)+4^2)

= (3x+4)(9x^2−12x+16)

(3x+4)(9x^2−12x+16) is the factorised form of 27x^3+64

what is 7/9 divided by 1/3​

Answers

Answer:

7/3 or 2.333333333

Step-by-step explanation:

Answer:

Answer is 2 1/3

Step-by-step explanation:

7/9 divided by 1/3 you can't divide fractions so you multiply so it would look more like:

7/9 * 1/3 = 7/3

you then divide 7 by 3 and get 2 1/3

So your answer is 2 1/3

Both the domain and range of the graph shown above is all real numbers?

Answers

Answer:

A::True

Step-by-step explanation:

For the quadratic function the domain is all real numbers since the horizontal extent of the graph is the whole real number line. Because the graph does not include any negative values for the range, the range is only non-negative real numbers.

Solve the equation.

–3x + 1 + 10x = x + 4

Answers

Answer:

x= 0.5

Step-by-step explanation:

It makes sense

The value of x is 1/2.

What is Equation?

Equations are mathematical statements with two algebraic expressions flanking the equals (=) sign on either side.

It demonstrates the equality of the relationship between the expressions printed on the left and right sides. LHS = RHS is a common mathematical formula.

Coefficients, variables, operators, constants, terms, expressions, and the equal to sign are some of the components of an equation. The "=" sign and terms on both sides must always be present when writing an equation.

Given:

–3x + 1 + 10x = x + 4

Now, solving for x

-3x+ 10x - x = 4 - 1

7x - x = 3

6x = 3

x= 3/6

x= 1/2

Hence, the value of x is 1/2.

Learn more about Equation here:

https://brainly.com/question/29657983

#SPJ2

Is x^2+8x+16 a perfect square

Answers

Answer:

YES IT IS A PERFECT SQUARE

Step-by-step explanation:

equate the equation to zero

x^2+8x+16=0

c is the product while the coefficient of b is the sum

P=16

S=8

find a pair of numbers that the product is 16 and the sum is 8

the numbers are (4,4)

(x^2+4x) (4X+16)

ignore 8x and arrange as shown above

then bracket the equation

x(x+4)+4(x+4)

you simplify and remain with (x+4) (x+4)

having similar roots making it a perfect square

Jim drove 756 miles in 12 hours.
At the same rate, how many miles would he drive in 7 hours?

Answers

Answer:

441

Step-by-step explanation:

Use a proportion.

756 miles is to 12 hours as x miles is to 7 hours

756/12 = x/7

12x = 7 * 756

12x = 5292

x = 441

Answer: 441 miles

Answer:

441

Step-by-step explanation:

756 divided by 12 = 63

Jim drives 63 miles in 1 hour

Multiply 63 by 7

63 ⋅ 7 = 441

Jim will drive 441 miles in 7 hours

Hope I helped :)

Please consider Brainliest :)

Jon caught four fish that weighed a total of 276 pounds. The kingfish weighed twice as much as the amberjack and the white marlin weighed twice as much as the kingfish. The weight of the tarpon was 5 times the weight of the amberjack.

How much did each fish weigh?

Answers

Answer:

amberjack 21 pounds

kingfish 42 pounds

marlin 84 pounds

tarpon 105 pounds

Step-by-step explanation:

The weight of the fish is given below,

The weight of the Kingfish is 20 pounds.The weight of the amberjack fish is 40 pounds.The weight of the white marlin fish is 10 pounds.The weight of the tarpon fish is 200 pounds.

What is an expression?

The mathematical expression combines numerical variables and operations denoted by addition, subtraction, multiplication, and division signs.

Mathematical symbols can be used to represent numbers (constants), variables, operations, functions, brackets, punctuation, and grouping. They can also denote the logical syntax's operation order and other properties.

Given that Jon caught four fish that weighed a total of 276 pounds. The kingfish weighed twice as much as the amberjack and the white marlin weighed twice as much as the kingfish. The weight of the tarpon was 5 times the weight of the amberjack.

The equation can be written as,

KF+A+W+T=276

Other relations are,

KF = A / 2

W = A /4

T = 5A

Substitute all the values in the above equation and solve for A.

KF+A+W+T=276

A / 2 + A + A / 4 + 5A = 276

2A + 4A +A +20A = 276 x 4

27A = 1104

A = 40 pounds

kingfish = KF = A / 2 = 20 pounds

White marlin = W = A / 4 = 10 pounds

Tarpon = T = 5A = 200 pounds

To know more about an expression follow

https://brainly.com/question/18564877

#SPJ2

Other Questions
can ANYONE PLEASE give me a fairy tale/folk tale that they have wrote before?? It has to be 3 pages long!! I will venmo money if needed!!!!!!!! PLEASE HELP ME!!! THANK YOUUU Which is the best answer? What are mound builders know for? Benjamn acept el puesto de camarero, pero l todava no ___ ___ ___________ (averiguar/ presentarse) en el restaurante.In present perfect for please Helpppp for forensic science I need to know what a cup would be classified as evidence wise. options biological, physical, impression or trace evidence. The consequence that an element of society produces for the maintenance of its social system. Which of the following pairs of numbers have a greatest common factor of 6?Select all that apply.A 6 and 3B 30 and 36C 24 and 48D 60 and 88E 18 and 66F 72 and 84 function rule y=3x-3 x + 5 > 7its inequalities if c= 2m + d ,Wich equation represents m The nearby residents questioned why theboat (that was docked at the end of the pier)remained there through the winter whenother boats usually left for warmer waters.Which edit should be made to correct thissentence? An aqueous solution of Pb(NO3)2 is made by placing 275 g of solid Pb(NO3)2 into a volumetric flask and adding water to the 1.00 L mark (assume that 775g of water has been added to achieve this total solution volume). (Assume MW of Pb(NO3)2 = 331g/mole) A) What is the molarity (M) of this solution? B) What is the molality (m) of this solution? C) What is the mass % of Pb(NO3)2 in this solution? D) What is the mole fraction of Pb(NO3)2 present in this solution? can someone help simplify this 6n+5(10n7) What is the slope of the line that passes through the points (-8, -6) and (-18, 6) ? Write your answer in simplest form. The amount of W water used varies directly with the population N of people. City A has a population of 468,000 people and used 37.6 billion gallons of water we would expect City B to use if we know that it's population is 180,000. James went to the amusement park. He had $20. He spent $5.50 on a hamburger and$1.25 on a bottle of apple juice. He spent the rest of the money on rides. Each ride costs$.75. On how many rides did James go?Please its urgent, write the steps. The Function of Fireworks Summary Can anyone tell me what the correct answer for this is? Thank you. Slove 4(5x-2)= 2(9x+3) .***9. The game of euchre uses only the 9s, 10s, it isjacks, queens, kings, and aces from a standarddeck of cards. How many five-card hands havea) all red cards?b) at least two red cards?c) at most two red cards? Answer a) From those information we know that have 24 cardIn those cards it have 12 red.12C5=792B) at least 2 red card=No restriction- without red card- at least one red card= 24C5-(12C0*12C5)-(12C1*12C4)=35772C) at most 2 red card 24C5-(12C0*12C5)-(12C1*12C4)-(12C2*12C3)=21252